Page 1 of 1

PT 25 - Section 4 #7 LR

Posted: Sun Mar 06, 2011 5:39 pm
by darkatillam2
Ok, I really need some help with this one.


So I know how A is right, but have absolutely 0 clue on why the answer I choose (C) is wrong.

As far as I can tell, both A and C are correct.

C is saying the net supply of fish will decrease, which will cause demand of fish to increase and prices to rise.

How is this not a supported conclusion based on how C and the stimulus is worded? Someone please help!

Re: PT 25 - Section 4 #7 LR

Posted: Sun Mar 06, 2011 10:12 pm
by benito
Your mistake is in confusing the market for fish in general with that for halibut specifically . The reason C doesn't justify the conclusion is, whether or not there would be an increase in the production of other kinds of fish does not effect the supply/demand dynamic for halibut. If the same # of people want halibut but the supply of it is decreased the price will increase whether or not there is more tuna or salmon or whatever on the market then there used to be.